Q5

 
jaydizzle
Thanks Received: 0
Vinny Gambini
Vinny Gambini
 
Posts: 14
Joined: January 10th, 2010
 
 
 

Q5

by jaydizzle Thu Jun 03, 2010 9:43 pm

I can't get question 5 right. If I put L in 3 and L in 9th, this would give me the maximum separation?

Wouldn't JKLJMKMOL work?
User avatar
 
ManhattanPrepLSAT1
Thanks Received: 1909
Atticus Finch
Atticus Finch
 
Posts: 2851
Joined: October 07th, 2009
 
 
 

Re: PT34, S4,G1 - Jill, Kurt, Larisa, Manny, and Olga

by ManhattanPrepLSAT1 Fri Jun 04, 2010 1:40 pm

Sure. You're hypothetical would work, but that would not separate Larissa's aisles with the maximum number of aisles.

Larrisa's aisles cannot be on both aisles 1 and 9. So, bring them in just a bit. 2 and 9 won't work, because K is assigned to the second aisle, but 1 and 8 would work using the scenario above.
Here are the possible scenarios

The MKM chunk must go either 3, 4, and 5 or must go 4, 5, and 6.

L K M K M O J L J
L K M K M J O L J
L K J M K M O L J
1 2 3 4 5 6 7 8 9

Here you can see that answer choice (A) could be true...

Does that clear it up? Can you see how the scenarios were generated? On questions that ask for the maximum, start with the maximum, which in this case would be putting Larissa on aisles 1 and 9. Since that doesn't work, bring her aisles in just a bit, so don't go down to 1 and 5, try 1 and 8 or 2 and 9.
 
jaydizzle
Thanks Received: 0
Vinny Gambini
Vinny Gambini
 
Posts: 14
Joined: January 10th, 2010
 
 
 

Re: PT34, S4,G1 - Jill, Kurt, Larisa, Manny, and Olga

by jaydizzle Fri Jun 04, 2010 2:16 pm

Oh boy, no wonder why! haha I was looking at the answer choices thinking, gee there are 2 or 3 that or correct what the hell? This is probably why I am stuck in the 150s. :(